狄拉克δ函数的导数

原文见 Physics Pages

狄拉克δ函数的图像像个钉子,如下图所示,谈论他的导数好像比较奇怪。



δ函数

我们从狄拉克δ函数的积分性质开始它的导数。狄拉克δ函数具有如下性质:

\begin{equation} \int_{-\infty}^{\infty} f(x)\delta(x)\mathrm dx=f(0) \tag{1}\label{eq1} \end{equation}

狄拉克δ函数的$n$阶导数为$\delta^{(n)}(x)$,做如下分部积分

\begin{equation}
\int_{-\infty}^{\infty} f(x)\delta^{(n)}(x)\mathrm dx=f(x)\delta^{(n-1)}(x)\big\lvert_{-\infty}^{\infty}-\int_{-\infty}^{\infty} f'(x)\delta^{(n-1)}(x)\mathrm dx
\tag{2}\label{eq2}
\end{equation}

第一项是0,因为狄拉克δ函数在$x\neq 0$的地方是常数0,因此导数也为0。于是我们有

\begin{equation}
\int_{-\infty}^{\infty} f(x)\delta^{(n)}(x)\mathrm dx=-\int_{-\infty}^{\infty} f'(x)\delta^{(n-1)}(x)\mathrm dx
\tag{3}\label{eq3}
\end{equation}

上式对任意函数$f(x)$都成立,因此两边被积函数相等,

\begin{equation}
f(x)\delta^{(n)}(x) =- f'(x)\delta^{(n-1)}(x)
\tag{4}\label{eq4}
\end{equation}

对于一阶导数有

\begin{equation}
f(x)\delta'(x) =- f'(x)\delta(x)
\tag{5}\label{eq5}
\end{equation}

如果$f(x)=x$,有

\begin{equation}
x\delta'(x) =- \delta(x)
\tag{6}\label{eq6}
\end{equation}

将\eqref{eq4}迭代下去,得

\begin{equation}
f(x)\delta^{(n)}(x) =(-1)^n \delta(x)\prod_{k=1}^nf^{(k)}(x)
\tag{7}\label{eq7}
\end{equation}

例1 令$f(x)=4x^2-1$,有

\begin{equation}
\int_{-\infty}^{\infty} (4x^2-1)\delta'(x-3)\mathrm dx=-\int_{-\infty}^{\infty} 8x\delta(x-3)\mathrm dx=-24
\tag{8}\label{eq8}
\end{equation}

例2 令$f(x)=x^n$,由\eqref{eq7}式有

\begin{equation}
x^n\delta^{(n)}(x) =(-1)^n n!\delta(x)
\tag{9}\label{eq9}
\end{equation}

标签: 狄拉克δ函数, 导数, 分部积分

已有 13 条评论

  1. wjl wjl

    博主您好!第7式的连乘应该是错的吧!我们可以直接推导一个函数与\delta函数二阶导数的积分,没有理由会出现连乘吧!

    1. 这篇文章问题太多了……
      别看了

  2. xinyue xinyue

    请问(7)式为什么不是直接为函数f(x)的n阶倒数,前面为什还有一个还求直积呀

  3. dao dao

    第一项是0,因为狄拉克δ函数在x≠0的地方是常数0,因此导数也为0,这里有问题吧

  4. mxlkevin mxlkevin

    根据(4)式考虑一阶和二阶导数的关系,另f(x)=x,同样会得到一阶导数与二阶导数间差-x这一个因子,那么原狄拉克函数和其二阶导数就只差x^2这一个因子,没有2!这一项。由(4)式到(7)式貌似确实不会出现阶乘项吧。

  5. 徐胤禛 徐胤禛

    在https://physics.stackexchange.com/questions/86041/peskin-and-schroeder-equation-2-56中提到公式5只有在f(0)=0时才是成立的,这个说法是否正确呢?

    1. 幽谷之草 幽谷之草

      按照广义函数的定义,这里应该要求f在无穷远处的极限是0,而不是f(0)=0

    2. 我感觉你发的链接里说的不对

  6. Laure Laure

    第一步分部积分处“第一项是0,因为狄拉克δ函数在x≠0的地方是常数0,因此导数也为0。”这不直接用了狄拉克函数导数为零?

    1. 你说的对啊。容我再想想。

      1. 宇

        请问公式结论现在可以直接用么?或者在什么限制条件下是成立的?

        1. 最后的结论是对的。

添加新评论

captcha
请输入验证码